Difference between revisions of "1960 AHSME Problems/Problem 30"

(Solution to Problem 30)
 
m (See Also)
 
Line 17: Line 17:
 
==See Also==
 
==See Also==
 
{{AHSME 40p box|year=1960|num-b=29|num-a=31}}
 
{{AHSME 40p box|year=1960|num-b=29|num-a=31}}
 +
 +
[[Category:Introductory Algebra Problems]]

Latest revision as of 19:13, 17 May 2018

Problem

Given the line $3x+5y=15$ and a point on this line equidistant from the coordinate axes. Such a point exists in:

$\textbf{(A)}\ \text{none of the quadrants}\qquad\textbf{(B)}\ \text{quadrant I only}\qquad\textbf{(C)}\ \text{quadrants I, II only}\qquad$ $\textbf{(D)}\ \text{quadrants I, II, III only}\qquad\textbf{(E)}\ \text{each of the quadrants}$

Solution

If a point is equidistant from the coordinate axes, then the absolute values of the x-coordinate and y-coordinate are equal. Since the point is on the line $3x+5y=15$, find the intersection point of $y=x$ and $3x+5y=15$ and the intersection point of $y=-x$ and $3x+5y=15$.

Substituting $x$ for $y$ in $3x+5y=15$ results in $8x=15$, so $x=\frac{15}{8}$. That means $y=\frac{15}{8}$, so one of the points is in the first quadrant.

Substituting $-x$ and $y$ in $3x+5y=15$ results in $-2x=15$, so $x=\frac{-15}{2}$. That means $y=\frac{15}{2}$, so the other point is in the second quadrant.

Thus, the points are in quadrants $I$ and $II$, so the answer is $\boxed{\textbf{(C)}}$.

See Also

1960 AHSC (ProblemsAnswer KeyResources)
Preceded by
Problem 29
Followed by
Problem 31
1 2 3 4 5 6 7 8 9 10 11 12 13 14 15 16 17 18 19 20 21 22 23 24 25 26 27 28 29 30 31 32 33 34 35 36 37 38 39 40
All AHSME Problems and Solutions